Quick but not simple question. $2^sqrt2$ or e, which is greater?Which of these numbers is greater: $sqrt[5]5$ or $sqrt[4]4$?Inequality through calculusWhich is greater, $300 !$ or $(300^300)^frac 12$?Hausdorff metric and Vietoris topologyClosed and Bounded but not compactWhich coefficient is greater?Calculating square roots using the recurrence $x_n+1 = frac12 left(x_n + frac2x_nright)$Trigonometric inequality - x domain$e^left(pi^(e^pi)right);$ or $;pi^left(e^(pi^e)right)$. Which one is greater than the other?Prove by contradiction (not using a calculator) that $sqrt6 + sqrt2 < sqrt15$?

Dot above capital letter not centred

What to do with wrong results in talks?

Is there an Impartial Brexit Deal comparison site?

How do I define a right arrow with bar in LaTeX?

Can I Retrieve Email Addresses from BCC?

Lay out the Carpet

Failed to fetch jessie backports repository

What are the ramifications of creating a homebrew world without an Astral Plane?

Everything Bob says is false. How does he get people to trust him?

Why "be dealt cards" rather than "be dealing cards"?

Trouble understanding overseas colleagues

Is a roofing delivery truck likely to crack my driveway slab?

What is the intuitive meaning of having a linear relationship between the logs of two variables?

What defines a dissertation?

What would be the benefits of having both a state and local currencies?

There is only s̶i̶x̶t̶y one place he can be

Bash method for viewing beginning and end of file

voltage of sounds of mp3files

Is it okay / does it make sense for another player to join a running game of Munchkin?

Modify casing of marked letters

I'm in charge of equipment buying but no one's ever happy with what I choose. How to fix this?

Was Spock the First Vulcan in Starfleet?

Teaching indefinite integrals that require special-casing

Hostile work environment after whistle-blowing on coworker and our boss. What do I do?



Quick but not simple question. $2^sqrt2$ or e, which is greater?


Which of these numbers is greater: $sqrt[5]5$ or $sqrt[4]4$?Inequality through calculusWhich is greater, $300 !$ or $(300^300)^frac 12$?Hausdorff metric and Vietoris topologyClosed and Bounded but not compactWhich coefficient is greater?Calculating square roots using the recurrence $x_n+1 = frac12 left(x_n + frac2x_nright)$Trigonometric inequality - x domain$e^left(pi^(e^pi)right);$ or $;pi^left(e^(pi^e)right)$. Which one is greater than the other?Prove by contradiction (not using a calculator) that $sqrt6 + sqrt2 < sqrt15$?













5












$begingroup$



$2^sqrt2$ vs $e$, which is greater?




$(2^sqrt2)^sqrt2 = 4quad $ & $quad e^sqrt2$ = ?



$log(2^sqrt2) = sqrt2log(2)quad$ & $quad log(e) = 1$



I tried but can't induce comparable form.



Is anybody know how to prove it?










share|cite|improve this question











$endgroup$











  • $begingroup$
    What is your question?
    $endgroup$
    – Michael Rozenberg
    yesterday










  • $begingroup$
    2^√2 > e or 2^√2 < e ? is my question
    $endgroup$
    – J.Bo
    yesterday







  • 3




    $begingroup$
    Well, the solution to $2^x=e$ is $x=frac 1ln 2approx 1.4427>sqrt 2$. Of course, numerical computation is involved in that.
    $endgroup$
    – lulu
    yesterday






  • 3




    $begingroup$
    What's wrong with using a calculator?
    $endgroup$
    – fleablood
    yesterday






  • 1




    $begingroup$
    Oh, that's simple but good idea. THX
    $endgroup$
    – J.Bo
    yesterday















5












$begingroup$



$2^sqrt2$ vs $e$, which is greater?




$(2^sqrt2)^sqrt2 = 4quad $ & $quad e^sqrt2$ = ?



$log(2^sqrt2) = sqrt2log(2)quad$ & $quad log(e) = 1$



I tried but can't induce comparable form.



Is anybody know how to prove it?










share|cite|improve this question











$endgroup$











  • $begingroup$
    What is your question?
    $endgroup$
    – Michael Rozenberg
    yesterday










  • $begingroup$
    2^√2 > e or 2^√2 < e ? is my question
    $endgroup$
    – J.Bo
    yesterday







  • 3




    $begingroup$
    Well, the solution to $2^x=e$ is $x=frac 1ln 2approx 1.4427>sqrt 2$. Of course, numerical computation is involved in that.
    $endgroup$
    – lulu
    yesterday






  • 3




    $begingroup$
    What's wrong with using a calculator?
    $endgroup$
    – fleablood
    yesterday






  • 1




    $begingroup$
    Oh, that's simple but good idea. THX
    $endgroup$
    – J.Bo
    yesterday













5












5








5


2



$begingroup$



$2^sqrt2$ vs $e$, which is greater?




$(2^sqrt2)^sqrt2 = 4quad $ & $quad e^sqrt2$ = ?



$log(2^sqrt2) = sqrt2log(2)quad$ & $quad log(e) = 1$



I tried but can't induce comparable form.



Is anybody know how to prove it?










share|cite|improve this question











$endgroup$





$2^sqrt2$ vs $e$, which is greater?




$(2^sqrt2)^sqrt2 = 4quad $ & $quad e^sqrt2$ = ?



$log(2^sqrt2) = sqrt2log(2)quad$ & $quad log(e) = 1$



I tried but can't induce comparable form.



Is anybody know how to prove it?







real-analysis analysis inequality






share|cite|improve this question















share|cite|improve this question













share|cite|improve this question




share|cite|improve this question








edited yesterday









YuiTo Cheng

2,1362837




2,1362837










asked yesterday









J.BoJ.Bo

456




456











  • $begingroup$
    What is your question?
    $endgroup$
    – Michael Rozenberg
    yesterday










  • $begingroup$
    2^√2 > e or 2^√2 < e ? is my question
    $endgroup$
    – J.Bo
    yesterday







  • 3




    $begingroup$
    Well, the solution to $2^x=e$ is $x=frac 1ln 2approx 1.4427>sqrt 2$. Of course, numerical computation is involved in that.
    $endgroup$
    – lulu
    yesterday






  • 3




    $begingroup$
    What's wrong with using a calculator?
    $endgroup$
    – fleablood
    yesterday






  • 1




    $begingroup$
    Oh, that's simple but good idea. THX
    $endgroup$
    – J.Bo
    yesterday
















  • $begingroup$
    What is your question?
    $endgroup$
    – Michael Rozenberg
    yesterday










  • $begingroup$
    2^√2 > e or 2^√2 < e ? is my question
    $endgroup$
    – J.Bo
    yesterday







  • 3




    $begingroup$
    Well, the solution to $2^x=e$ is $x=frac 1ln 2approx 1.4427>sqrt 2$. Of course, numerical computation is involved in that.
    $endgroup$
    – lulu
    yesterday






  • 3




    $begingroup$
    What's wrong with using a calculator?
    $endgroup$
    – fleablood
    yesterday






  • 1




    $begingroup$
    Oh, that's simple but good idea. THX
    $endgroup$
    – J.Bo
    yesterday















$begingroup$
What is your question?
$endgroup$
– Michael Rozenberg
yesterday




$begingroup$
What is your question?
$endgroup$
– Michael Rozenberg
yesterday












$begingroup$
2^√2 > e or 2^√2 < e ? is my question
$endgroup$
– J.Bo
yesterday





$begingroup$
2^√2 > e or 2^√2 < e ? is my question
$endgroup$
– J.Bo
yesterday





3




3




$begingroup$
Well, the solution to $2^x=e$ is $x=frac 1ln 2approx 1.4427>sqrt 2$. Of course, numerical computation is involved in that.
$endgroup$
– lulu
yesterday




$begingroup$
Well, the solution to $2^x=e$ is $x=frac 1ln 2approx 1.4427>sqrt 2$. Of course, numerical computation is involved in that.
$endgroup$
– lulu
yesterday




3




3




$begingroup$
What's wrong with using a calculator?
$endgroup$
– fleablood
yesterday




$begingroup$
What's wrong with using a calculator?
$endgroup$
– fleablood
yesterday




1




1




$begingroup$
Oh, that's simple but good idea. THX
$endgroup$
– J.Bo
yesterday




$begingroup$
Oh, that's simple but good idea. THX
$endgroup$
– J.Bo
yesterday










4 Answers
4






active

oldest

votes


















8












$begingroup$

Instead of comparing $2^sqrt 2$ and $e$, let's raise both to $sqrt 2$ and compare $2^2$ and $e^sqrt 2$:
$$
e^sqrt 2 > 2.7^1.4 approx 4.017068799 > 4 = 2^2
$$

Or use that
$$
e^x > 1+x+fracx^22+fracx^36+fracx^424
$$

with $x=1.41$ and get
$$
e^sqrt 2 > e^1.41 > 4.03594 > 4
$$

In fact, $e^sqrt 2 approx 4.113250377 > 4$.






share|cite|improve this answer









$endgroup$








  • 5




    $begingroup$
    I don't understand how you can estimate $2.7^1.4$, $e^1.41$, and $e^sqrt 2$ without a calculator. And if you have a calculator, why not find $2^sqrt 2$ from the beginning?
    $endgroup$
    – Teepeemm
    yesterday










  • $begingroup$
    @Teepeemm, you're right. The best approach is the second one, with a polynomial. Unfortunately, it's of degree $4$ and you have to use two decimals in $x=1.41$.
    $endgroup$
    – lhf
    yesterday


















7












$begingroup$

This is the same as comparing $frac32log(2)$ and $1$. Since $x(1-x)$ is non-negative and bounded by $frac14$ on $(0,1)$, we have
$$ 0leqint_0^1fracx^2(1-x)^21+x,dx leq frac116$$
where the middle integral is exactly $-frac114+4log(2)$. It follows that
$$ frac3332 leq frac32log(2) leq frac135128 $$
so $frac32log(2)>1$ and $colorred2sqrt2>e$.
This proof just requires a polynomial division, perfectly doable by hand.



About $sqrt2log(2)$, we have
$$ log(2)=lim_nto +inftysum_k=n+1^2nfrac1kleqlim_nto+inftysum_k=n+1^2nfrac1sqrtksqrtk-1stackreltextCSleqlim_nto +inftysqrtnsum_k=n+1^2nleft(frac1k-1-frac1kright)$$
and the RHS is exactly $frac1sqrt2$. This is just a slick application of creative telescoping and the Cauchy-Schwarz inequality.






share|cite|improve this answer











$endgroup$












  • $begingroup$
    It's $2^sqrt2$. See here
    $endgroup$
    – YuiTo Cheng
    yesterday











  • $begingroup$
    @YuiToCheng: well, I dealt with both cases.
    $endgroup$
    – Jack D'Aurizio
    yesterday






  • 1




    $begingroup$
    +1. I think your answer truly doesn't require any numerical calculation.
    $endgroup$
    – YuiTo Cheng
    yesterday


















5












$begingroup$

If you know that $ln(2)approx0.69$ and $1/sqrt2=sqrt2/2approx1.4.14/2=0.707$, then you have $ln(2)lt1/sqrt2$, in which case $ln(2^sqrt2)=sqrt2ln2lt1=ln(e)$, hence $2^sqrt2lt e$.



It's not hard to show that $sqrt2gt1.4$, since $1.4^2=1.96lt2$. It's a little trickier to show that $ln(2)lt0.7$, but this can be done by comparing the area beneath the curve $y=1/x$ to the areas of the trapezoids containing it with endpoints at $x=1$, $4/3$, $5/3$, and $2$:



$$ln(2)=int_1^2dxover xlt1over6left(1+2cdot3over4+2cdot3over5+1over2 right)=1over6left(1+3over2+6over5+1over2 right)=1over6cdot42over10=7over10$$






share|cite|improve this answer









$endgroup$








  • 2




    $begingroup$
    Nice solution because this does not require a calculator.
    $endgroup$
    – quarague
    yesterday


















0












$begingroup$

$2sqrt2^2 = 8$



$e^2 < 2.8*2.8 = 7.84$



You're welcome






share|cite|improve this answer








New contributor




Felor is a new contributor to this site. Take care in asking for clarification, commenting, and answering.
Check out our Code of Conduct.






$endgroup$












  • $begingroup$
    It's $2^sqrt2$...
    $endgroup$
    – YuiTo Cheng
    yesterday










  • $begingroup$
    Aww, there is a bug in EE. Yellow formula in startpost is $2sqrt2$ for me.
    $endgroup$
    – Felor
    yesterday











  • $begingroup$
    I see. It's not your fault. Corrected. It's a careless typo.
    $endgroup$
    – YuiTo Cheng
    yesterday











  • $begingroup$
    One can start with taylor of $e^x$ then. $1+x+...+fracx^5120$, where x is $1.4 < sqrt2$. Sum of it gives 4.042219. Can be calculated by hand. $1.4 < sqrt2$ is trivial.
    $endgroup$
    – Felor
    yesterday











  • $begingroup$
    Yeah, it's exactly the summary of the first answer.
    $endgroup$
    – YuiTo Cheng
    yesterday










Your Answer





StackExchange.ifUsing("editor", function ()
return StackExchange.using("mathjaxEditing", function ()
StackExchange.MarkdownEditor.creationCallbacks.add(function (editor, postfix)
StackExchange.mathjaxEditing.prepareWmdForMathJax(editor, postfix, [["$", "$"], ["\\(","\\)"]]);
);
);
, "mathjax-editing");

StackExchange.ready(function()
var channelOptions =
tags: "".split(" "),
id: "69"
;
initTagRenderer("".split(" "), "".split(" "), channelOptions);

StackExchange.using("externalEditor", function()
// Have to fire editor after snippets, if snippets enabled
if (StackExchange.settings.snippets.snippetsEnabled)
StackExchange.using("snippets", function()
createEditor();
);

else
createEditor();

);

function createEditor()
StackExchange.prepareEditor(
heartbeatType: 'answer',
autoActivateHeartbeat: false,
convertImagesToLinks: true,
noModals: true,
showLowRepImageUploadWarning: true,
reputationToPostImages: 10,
bindNavPrevention: true,
postfix: "",
imageUploader:
brandingHtml: "Powered by u003ca class="icon-imgur-white" href="https://imgur.com/"u003eu003c/au003e",
contentPolicyHtml: "User contributions licensed under u003ca href="https://creativecommons.org/licenses/by-sa/3.0/"u003ecc by-sa 3.0 with attribution requiredu003c/au003e u003ca href="https://stackoverflow.com/legal/content-policy"u003e(content policy)u003c/au003e",
allowUrls: true
,
noCode: true, onDemand: true,
discardSelector: ".discard-answer"
,immediatelyShowMarkdownHelp:true
);



);













draft saved

draft discarded


















StackExchange.ready(
function ()
StackExchange.openid.initPostLogin('.new-post-login', 'https%3a%2f%2fmath.stackexchange.com%2fquestions%2f3161742%2fquick-but-not-simple-question-2-sqrt2-or-e-which-is-greater%23new-answer', 'question_page');

);

Post as a guest















Required, but never shown

























4 Answers
4






active

oldest

votes








4 Answers
4






active

oldest

votes









active

oldest

votes






active

oldest

votes









8












$begingroup$

Instead of comparing $2^sqrt 2$ and $e$, let's raise both to $sqrt 2$ and compare $2^2$ and $e^sqrt 2$:
$$
e^sqrt 2 > 2.7^1.4 approx 4.017068799 > 4 = 2^2
$$

Or use that
$$
e^x > 1+x+fracx^22+fracx^36+fracx^424
$$

with $x=1.41$ and get
$$
e^sqrt 2 > e^1.41 > 4.03594 > 4
$$

In fact, $e^sqrt 2 approx 4.113250377 > 4$.






share|cite|improve this answer









$endgroup$








  • 5




    $begingroup$
    I don't understand how you can estimate $2.7^1.4$, $e^1.41$, and $e^sqrt 2$ without a calculator. And if you have a calculator, why not find $2^sqrt 2$ from the beginning?
    $endgroup$
    – Teepeemm
    yesterday










  • $begingroup$
    @Teepeemm, you're right. The best approach is the second one, with a polynomial. Unfortunately, it's of degree $4$ and you have to use two decimals in $x=1.41$.
    $endgroup$
    – lhf
    yesterday















8












$begingroup$

Instead of comparing $2^sqrt 2$ and $e$, let's raise both to $sqrt 2$ and compare $2^2$ and $e^sqrt 2$:
$$
e^sqrt 2 > 2.7^1.4 approx 4.017068799 > 4 = 2^2
$$

Or use that
$$
e^x > 1+x+fracx^22+fracx^36+fracx^424
$$

with $x=1.41$ and get
$$
e^sqrt 2 > e^1.41 > 4.03594 > 4
$$

In fact, $e^sqrt 2 approx 4.113250377 > 4$.






share|cite|improve this answer









$endgroup$








  • 5




    $begingroup$
    I don't understand how you can estimate $2.7^1.4$, $e^1.41$, and $e^sqrt 2$ without a calculator. And if you have a calculator, why not find $2^sqrt 2$ from the beginning?
    $endgroup$
    – Teepeemm
    yesterday










  • $begingroup$
    @Teepeemm, you're right. The best approach is the second one, with a polynomial. Unfortunately, it's of degree $4$ and you have to use two decimals in $x=1.41$.
    $endgroup$
    – lhf
    yesterday













8












8








8





$begingroup$

Instead of comparing $2^sqrt 2$ and $e$, let's raise both to $sqrt 2$ and compare $2^2$ and $e^sqrt 2$:
$$
e^sqrt 2 > 2.7^1.4 approx 4.017068799 > 4 = 2^2
$$

Or use that
$$
e^x > 1+x+fracx^22+fracx^36+fracx^424
$$

with $x=1.41$ and get
$$
e^sqrt 2 > e^1.41 > 4.03594 > 4
$$

In fact, $e^sqrt 2 approx 4.113250377 > 4$.






share|cite|improve this answer









$endgroup$



Instead of comparing $2^sqrt 2$ and $e$, let's raise both to $sqrt 2$ and compare $2^2$ and $e^sqrt 2$:
$$
e^sqrt 2 > 2.7^1.4 approx 4.017068799 > 4 = 2^2
$$

Or use that
$$
e^x > 1+x+fracx^22+fracx^36+fracx^424
$$

with $x=1.41$ and get
$$
e^sqrt 2 > e^1.41 > 4.03594 > 4
$$

In fact, $e^sqrt 2 approx 4.113250377 > 4$.







share|cite|improve this answer












share|cite|improve this answer



share|cite|improve this answer










answered yesterday









lhflhf

167k11172403




167k11172403







  • 5




    $begingroup$
    I don't understand how you can estimate $2.7^1.4$, $e^1.41$, and $e^sqrt 2$ without a calculator. And if you have a calculator, why not find $2^sqrt 2$ from the beginning?
    $endgroup$
    – Teepeemm
    yesterday










  • $begingroup$
    @Teepeemm, you're right. The best approach is the second one, with a polynomial. Unfortunately, it's of degree $4$ and you have to use two decimals in $x=1.41$.
    $endgroup$
    – lhf
    yesterday












  • 5




    $begingroup$
    I don't understand how you can estimate $2.7^1.4$, $e^1.41$, and $e^sqrt 2$ without a calculator. And if you have a calculator, why not find $2^sqrt 2$ from the beginning?
    $endgroup$
    – Teepeemm
    yesterday










  • $begingroup$
    @Teepeemm, you're right. The best approach is the second one, with a polynomial. Unfortunately, it's of degree $4$ and you have to use two decimals in $x=1.41$.
    $endgroup$
    – lhf
    yesterday







5




5




$begingroup$
I don't understand how you can estimate $2.7^1.4$, $e^1.41$, and $e^sqrt 2$ without a calculator. And if you have a calculator, why not find $2^sqrt 2$ from the beginning?
$endgroup$
– Teepeemm
yesterday




$begingroup$
I don't understand how you can estimate $2.7^1.4$, $e^1.41$, and $e^sqrt 2$ without a calculator. And if you have a calculator, why not find $2^sqrt 2$ from the beginning?
$endgroup$
– Teepeemm
yesterday












$begingroup$
@Teepeemm, you're right. The best approach is the second one, with a polynomial. Unfortunately, it's of degree $4$ and you have to use two decimals in $x=1.41$.
$endgroup$
– lhf
yesterday




$begingroup$
@Teepeemm, you're right. The best approach is the second one, with a polynomial. Unfortunately, it's of degree $4$ and you have to use two decimals in $x=1.41$.
$endgroup$
– lhf
yesterday











7












$begingroup$

This is the same as comparing $frac32log(2)$ and $1$. Since $x(1-x)$ is non-negative and bounded by $frac14$ on $(0,1)$, we have
$$ 0leqint_0^1fracx^2(1-x)^21+x,dx leq frac116$$
where the middle integral is exactly $-frac114+4log(2)$. It follows that
$$ frac3332 leq frac32log(2) leq frac135128 $$
so $frac32log(2)>1$ and $colorred2sqrt2>e$.
This proof just requires a polynomial division, perfectly doable by hand.



About $sqrt2log(2)$, we have
$$ log(2)=lim_nto +inftysum_k=n+1^2nfrac1kleqlim_nto+inftysum_k=n+1^2nfrac1sqrtksqrtk-1stackreltextCSleqlim_nto +inftysqrtnsum_k=n+1^2nleft(frac1k-1-frac1kright)$$
and the RHS is exactly $frac1sqrt2$. This is just a slick application of creative telescoping and the Cauchy-Schwarz inequality.






share|cite|improve this answer











$endgroup$












  • $begingroup$
    It's $2^sqrt2$. See here
    $endgroup$
    – YuiTo Cheng
    yesterday











  • $begingroup$
    @YuiToCheng: well, I dealt with both cases.
    $endgroup$
    – Jack D'Aurizio
    yesterday






  • 1




    $begingroup$
    +1. I think your answer truly doesn't require any numerical calculation.
    $endgroup$
    – YuiTo Cheng
    yesterday















7












$begingroup$

This is the same as comparing $frac32log(2)$ and $1$. Since $x(1-x)$ is non-negative and bounded by $frac14$ on $(0,1)$, we have
$$ 0leqint_0^1fracx^2(1-x)^21+x,dx leq frac116$$
where the middle integral is exactly $-frac114+4log(2)$. It follows that
$$ frac3332 leq frac32log(2) leq frac135128 $$
so $frac32log(2)>1$ and $colorred2sqrt2>e$.
This proof just requires a polynomial division, perfectly doable by hand.



About $sqrt2log(2)$, we have
$$ log(2)=lim_nto +inftysum_k=n+1^2nfrac1kleqlim_nto+inftysum_k=n+1^2nfrac1sqrtksqrtk-1stackreltextCSleqlim_nto +inftysqrtnsum_k=n+1^2nleft(frac1k-1-frac1kright)$$
and the RHS is exactly $frac1sqrt2$. This is just a slick application of creative telescoping and the Cauchy-Schwarz inequality.






share|cite|improve this answer











$endgroup$












  • $begingroup$
    It's $2^sqrt2$. See here
    $endgroup$
    – YuiTo Cheng
    yesterday











  • $begingroup$
    @YuiToCheng: well, I dealt with both cases.
    $endgroup$
    – Jack D'Aurizio
    yesterday






  • 1




    $begingroup$
    +1. I think your answer truly doesn't require any numerical calculation.
    $endgroup$
    – YuiTo Cheng
    yesterday













7












7








7





$begingroup$

This is the same as comparing $frac32log(2)$ and $1$. Since $x(1-x)$ is non-negative and bounded by $frac14$ on $(0,1)$, we have
$$ 0leqint_0^1fracx^2(1-x)^21+x,dx leq frac116$$
where the middle integral is exactly $-frac114+4log(2)$. It follows that
$$ frac3332 leq frac32log(2) leq frac135128 $$
so $frac32log(2)>1$ and $colorred2sqrt2>e$.
This proof just requires a polynomial division, perfectly doable by hand.



About $sqrt2log(2)$, we have
$$ log(2)=lim_nto +inftysum_k=n+1^2nfrac1kleqlim_nto+inftysum_k=n+1^2nfrac1sqrtksqrtk-1stackreltextCSleqlim_nto +inftysqrtnsum_k=n+1^2nleft(frac1k-1-frac1kright)$$
and the RHS is exactly $frac1sqrt2$. This is just a slick application of creative telescoping and the Cauchy-Schwarz inequality.






share|cite|improve this answer











$endgroup$



This is the same as comparing $frac32log(2)$ and $1$. Since $x(1-x)$ is non-negative and bounded by $frac14$ on $(0,1)$, we have
$$ 0leqint_0^1fracx^2(1-x)^21+x,dx leq frac116$$
where the middle integral is exactly $-frac114+4log(2)$. It follows that
$$ frac3332 leq frac32log(2) leq frac135128 $$
so $frac32log(2)>1$ and $colorred2sqrt2>e$.
This proof just requires a polynomial division, perfectly doable by hand.



About $sqrt2log(2)$, we have
$$ log(2)=lim_nto +inftysum_k=n+1^2nfrac1kleqlim_nto+inftysum_k=n+1^2nfrac1sqrtksqrtk-1stackreltextCSleqlim_nto +inftysqrtnsum_k=n+1^2nleft(frac1k-1-frac1kright)$$
and the RHS is exactly $frac1sqrt2$. This is just a slick application of creative telescoping and the Cauchy-Schwarz inequality.







share|cite|improve this answer














share|cite|improve this answer



share|cite|improve this answer








edited yesterday

























answered yesterday









Jack D'AurizioJack D'Aurizio

292k33284672




292k33284672











  • $begingroup$
    It's $2^sqrt2$. See here
    $endgroup$
    – YuiTo Cheng
    yesterday











  • $begingroup$
    @YuiToCheng: well, I dealt with both cases.
    $endgroup$
    – Jack D'Aurizio
    yesterday






  • 1




    $begingroup$
    +1. I think your answer truly doesn't require any numerical calculation.
    $endgroup$
    – YuiTo Cheng
    yesterday
















  • $begingroup$
    It's $2^sqrt2$. See here
    $endgroup$
    – YuiTo Cheng
    yesterday











  • $begingroup$
    @YuiToCheng: well, I dealt with both cases.
    $endgroup$
    – Jack D'Aurizio
    yesterday






  • 1




    $begingroup$
    +1. I think your answer truly doesn't require any numerical calculation.
    $endgroup$
    – YuiTo Cheng
    yesterday















$begingroup$
It's $2^sqrt2$. See here
$endgroup$
– YuiTo Cheng
yesterday





$begingroup$
It's $2^sqrt2$. See here
$endgroup$
– YuiTo Cheng
yesterday













$begingroup$
@YuiToCheng: well, I dealt with both cases.
$endgroup$
– Jack D'Aurizio
yesterday




$begingroup$
@YuiToCheng: well, I dealt with both cases.
$endgroup$
– Jack D'Aurizio
yesterday




1




1




$begingroup$
+1. I think your answer truly doesn't require any numerical calculation.
$endgroup$
– YuiTo Cheng
yesterday




$begingroup$
+1. I think your answer truly doesn't require any numerical calculation.
$endgroup$
– YuiTo Cheng
yesterday











5












$begingroup$

If you know that $ln(2)approx0.69$ and $1/sqrt2=sqrt2/2approx1.4.14/2=0.707$, then you have $ln(2)lt1/sqrt2$, in which case $ln(2^sqrt2)=sqrt2ln2lt1=ln(e)$, hence $2^sqrt2lt e$.



It's not hard to show that $sqrt2gt1.4$, since $1.4^2=1.96lt2$. It's a little trickier to show that $ln(2)lt0.7$, but this can be done by comparing the area beneath the curve $y=1/x$ to the areas of the trapezoids containing it with endpoints at $x=1$, $4/3$, $5/3$, and $2$:



$$ln(2)=int_1^2dxover xlt1over6left(1+2cdot3over4+2cdot3over5+1over2 right)=1over6left(1+3over2+6over5+1over2 right)=1over6cdot42over10=7over10$$






share|cite|improve this answer









$endgroup$








  • 2




    $begingroup$
    Nice solution because this does not require a calculator.
    $endgroup$
    – quarague
    yesterday















5












$begingroup$

If you know that $ln(2)approx0.69$ and $1/sqrt2=sqrt2/2approx1.4.14/2=0.707$, then you have $ln(2)lt1/sqrt2$, in which case $ln(2^sqrt2)=sqrt2ln2lt1=ln(e)$, hence $2^sqrt2lt e$.



It's not hard to show that $sqrt2gt1.4$, since $1.4^2=1.96lt2$. It's a little trickier to show that $ln(2)lt0.7$, but this can be done by comparing the area beneath the curve $y=1/x$ to the areas of the trapezoids containing it with endpoints at $x=1$, $4/3$, $5/3$, and $2$:



$$ln(2)=int_1^2dxover xlt1over6left(1+2cdot3over4+2cdot3over5+1over2 right)=1over6left(1+3over2+6over5+1over2 right)=1over6cdot42over10=7over10$$






share|cite|improve this answer









$endgroup$








  • 2




    $begingroup$
    Nice solution because this does not require a calculator.
    $endgroup$
    – quarague
    yesterday













5












5








5





$begingroup$

If you know that $ln(2)approx0.69$ and $1/sqrt2=sqrt2/2approx1.4.14/2=0.707$, then you have $ln(2)lt1/sqrt2$, in which case $ln(2^sqrt2)=sqrt2ln2lt1=ln(e)$, hence $2^sqrt2lt e$.



It's not hard to show that $sqrt2gt1.4$, since $1.4^2=1.96lt2$. It's a little trickier to show that $ln(2)lt0.7$, but this can be done by comparing the area beneath the curve $y=1/x$ to the areas of the trapezoids containing it with endpoints at $x=1$, $4/3$, $5/3$, and $2$:



$$ln(2)=int_1^2dxover xlt1over6left(1+2cdot3over4+2cdot3over5+1over2 right)=1over6left(1+3over2+6over5+1over2 right)=1over6cdot42over10=7over10$$






share|cite|improve this answer









$endgroup$



If you know that $ln(2)approx0.69$ and $1/sqrt2=sqrt2/2approx1.4.14/2=0.707$, then you have $ln(2)lt1/sqrt2$, in which case $ln(2^sqrt2)=sqrt2ln2lt1=ln(e)$, hence $2^sqrt2lt e$.



It's not hard to show that $sqrt2gt1.4$, since $1.4^2=1.96lt2$. It's a little trickier to show that $ln(2)lt0.7$, but this can be done by comparing the area beneath the curve $y=1/x$ to the areas of the trapezoids containing it with endpoints at $x=1$, $4/3$, $5/3$, and $2$:



$$ln(2)=int_1^2dxover xlt1over6left(1+2cdot3over4+2cdot3over5+1over2 right)=1over6left(1+3over2+6over5+1over2 right)=1over6cdot42over10=7over10$$







share|cite|improve this answer












share|cite|improve this answer



share|cite|improve this answer










answered yesterday









Barry CipraBarry Cipra

60.5k655128




60.5k655128







  • 2




    $begingroup$
    Nice solution because this does not require a calculator.
    $endgroup$
    – quarague
    yesterday












  • 2




    $begingroup$
    Nice solution because this does not require a calculator.
    $endgroup$
    – quarague
    yesterday







2




2




$begingroup$
Nice solution because this does not require a calculator.
$endgroup$
– quarague
yesterday




$begingroup$
Nice solution because this does not require a calculator.
$endgroup$
– quarague
yesterday











0












$begingroup$

$2sqrt2^2 = 8$



$e^2 < 2.8*2.8 = 7.84$



You're welcome






share|cite|improve this answer








New contributor




Felor is a new contributor to this site. Take care in asking for clarification, commenting, and answering.
Check out our Code of Conduct.






$endgroup$












  • $begingroup$
    It's $2^sqrt2$...
    $endgroup$
    – YuiTo Cheng
    yesterday










  • $begingroup$
    Aww, there is a bug in EE. Yellow formula in startpost is $2sqrt2$ for me.
    $endgroup$
    – Felor
    yesterday











  • $begingroup$
    I see. It's not your fault. Corrected. It's a careless typo.
    $endgroup$
    – YuiTo Cheng
    yesterday











  • $begingroup$
    One can start with taylor of $e^x$ then. $1+x+...+fracx^5120$, where x is $1.4 < sqrt2$. Sum of it gives 4.042219. Can be calculated by hand. $1.4 < sqrt2$ is trivial.
    $endgroup$
    – Felor
    yesterday











  • $begingroup$
    Yeah, it's exactly the summary of the first answer.
    $endgroup$
    – YuiTo Cheng
    yesterday















0












$begingroup$

$2sqrt2^2 = 8$



$e^2 < 2.8*2.8 = 7.84$



You're welcome






share|cite|improve this answer








New contributor




Felor is a new contributor to this site. Take care in asking for clarification, commenting, and answering.
Check out our Code of Conduct.






$endgroup$












  • $begingroup$
    It's $2^sqrt2$...
    $endgroup$
    – YuiTo Cheng
    yesterday










  • $begingroup$
    Aww, there is a bug in EE. Yellow formula in startpost is $2sqrt2$ for me.
    $endgroup$
    – Felor
    yesterday











  • $begingroup$
    I see. It's not your fault. Corrected. It's a careless typo.
    $endgroup$
    – YuiTo Cheng
    yesterday











  • $begingroup$
    One can start with taylor of $e^x$ then. $1+x+...+fracx^5120$, where x is $1.4 < sqrt2$. Sum of it gives 4.042219. Can be calculated by hand. $1.4 < sqrt2$ is trivial.
    $endgroup$
    – Felor
    yesterday











  • $begingroup$
    Yeah, it's exactly the summary of the first answer.
    $endgroup$
    – YuiTo Cheng
    yesterday













0












0








0





$begingroup$

$2sqrt2^2 = 8$



$e^2 < 2.8*2.8 = 7.84$



You're welcome






share|cite|improve this answer








New contributor




Felor is a new contributor to this site. Take care in asking for clarification, commenting, and answering.
Check out our Code of Conduct.






$endgroup$



$2sqrt2^2 = 8$



$e^2 < 2.8*2.8 = 7.84$



You're welcome







share|cite|improve this answer








New contributor




Felor is a new contributor to this site. Take care in asking for clarification, commenting, and answering.
Check out our Code of Conduct.









share|cite|improve this answer



share|cite|improve this answer






New contributor




Felor is a new contributor to this site. Take care in asking for clarification, commenting, and answering.
Check out our Code of Conduct.









answered yesterday









FelorFelor

1




1




New contributor




Felor is a new contributor to this site. Take care in asking for clarification, commenting, and answering.
Check out our Code of Conduct.





New contributor





Felor is a new contributor to this site. Take care in asking for clarification, commenting, and answering.
Check out our Code of Conduct.






Felor is a new contributor to this site. Take care in asking for clarification, commenting, and answering.
Check out our Code of Conduct.











  • $begingroup$
    It's $2^sqrt2$...
    $endgroup$
    – YuiTo Cheng
    yesterday










  • $begingroup$
    Aww, there is a bug in EE. Yellow formula in startpost is $2sqrt2$ for me.
    $endgroup$
    – Felor
    yesterday











  • $begingroup$
    I see. It's not your fault. Corrected. It's a careless typo.
    $endgroup$
    – YuiTo Cheng
    yesterday











  • $begingroup$
    One can start with taylor of $e^x$ then. $1+x+...+fracx^5120$, where x is $1.4 < sqrt2$. Sum of it gives 4.042219. Can be calculated by hand. $1.4 < sqrt2$ is trivial.
    $endgroup$
    – Felor
    yesterday











  • $begingroup$
    Yeah, it's exactly the summary of the first answer.
    $endgroup$
    – YuiTo Cheng
    yesterday
















  • $begingroup$
    It's $2^sqrt2$...
    $endgroup$
    – YuiTo Cheng
    yesterday










  • $begingroup$
    Aww, there is a bug in EE. Yellow formula in startpost is $2sqrt2$ for me.
    $endgroup$
    – Felor
    yesterday











  • $begingroup$
    I see. It's not your fault. Corrected. It's a careless typo.
    $endgroup$
    – YuiTo Cheng
    yesterday











  • $begingroup$
    One can start with taylor of $e^x$ then. $1+x+...+fracx^5120$, where x is $1.4 < sqrt2$. Sum of it gives 4.042219. Can be calculated by hand. $1.4 < sqrt2$ is trivial.
    $endgroup$
    – Felor
    yesterday











  • $begingroup$
    Yeah, it's exactly the summary of the first answer.
    $endgroup$
    – YuiTo Cheng
    yesterday















$begingroup$
It's $2^sqrt2$...
$endgroup$
– YuiTo Cheng
yesterday




$begingroup$
It's $2^sqrt2$...
$endgroup$
– YuiTo Cheng
yesterday












$begingroup$
Aww, there is a bug in EE. Yellow formula in startpost is $2sqrt2$ for me.
$endgroup$
– Felor
yesterday





$begingroup$
Aww, there is a bug in EE. Yellow formula in startpost is $2sqrt2$ for me.
$endgroup$
– Felor
yesterday













$begingroup$
I see. It's not your fault. Corrected. It's a careless typo.
$endgroup$
– YuiTo Cheng
yesterday





$begingroup$
I see. It's not your fault. Corrected. It's a careless typo.
$endgroup$
– YuiTo Cheng
yesterday













$begingroup$
One can start with taylor of $e^x$ then. $1+x+...+fracx^5120$, where x is $1.4 < sqrt2$. Sum of it gives 4.042219. Can be calculated by hand. $1.4 < sqrt2$ is trivial.
$endgroup$
– Felor
yesterday





$begingroup$
One can start with taylor of $e^x$ then. $1+x+...+fracx^5120$, where x is $1.4 < sqrt2$. Sum of it gives 4.042219. Can be calculated by hand. $1.4 < sqrt2$ is trivial.
$endgroup$
– Felor
yesterday













$begingroup$
Yeah, it's exactly the summary of the first answer.
$endgroup$
– YuiTo Cheng
yesterday




$begingroup$
Yeah, it's exactly the summary of the first answer.
$endgroup$
– YuiTo Cheng
yesterday

















draft saved

draft discarded
















































Thanks for contributing an answer to Mathematics Stack Exchange!


  • Please be sure to answer the question. Provide details and share your research!

But avoid


  • Asking for help, clarification, or responding to other answers.

  • Making statements based on opinion; back them up with references or personal experience.

Use MathJax to format equations. MathJax reference.


To learn more, see our tips on writing great answers.




draft saved


draft discarded














StackExchange.ready(
function ()
StackExchange.openid.initPostLogin('.new-post-login', 'https%3a%2f%2fmath.stackexchange.com%2fquestions%2f3161742%2fquick-but-not-simple-question-2-sqrt2-or-e-which-is-greater%23new-answer', 'question_page');

);

Post as a guest















Required, but never shown





















































Required, but never shown














Required, but never shown












Required, but never shown







Required, but never shown

































Required, but never shown














Required, but never shown












Required, but never shown







Required, but never shown







-analysis, inequality, real-analysis

Popular posts from this blog

Mobil Contents History Mobil brands Former Mobil brands Lukoil transaction Mobil UK Mobil Australia Mobil New Zealand Mobil Greece Mobil in Japan Mobil in Canada Mobil Egypt See also References External links Navigation menuwww.mobil.com"Mobil Corporation"the original"Our Houston campus""Business & Finance: Socony-Vacuum Corp.""Popular Mechanics""Lubrite Technologies""Exxon Mobil campus 'clearly happening'""Toledo Blade - Google News Archive Search""The Lion and the Moose - How 2 Executives Pulled off the Biggest Merger Ever""ExxonMobil Press Release""Lubricants""Archived copy"the original"Mobil 1™ and Mobil Super™ motor oil and synthetic motor oil - Mobil™ Motor Oils""Mobil Delvac""Mobil Industrial website""The State of Competition in Gasoline Marketing: The Effects of Refiner Operations at Retail""Mobil Travel Guide to become Forbes Travel Guide""Hotel Rankings: Forbes Merges with Mobil"the original"Jamieson oil industry history""Mobil news""Caltex pumps for control""Watchdog blocks Caltex bid""Exxon Mobil sells service station network""Mobil Oil New Zealand Limited is New Zealand's oldest oil company, with predecessor companies having first established a presence in the country in 1896""ExxonMobil subsidiaries have a business history in New Zealand stretching back more than 120 years. We are involved in petroleum refining and distribution and the marketing of fuels, lubricants and chemical products""Archived copy"the original"Exxon Mobil to Sell Its Japanese Arm for $3.9 Billion""Gas station merger will end Esso and Mobil's long run in Japan""Esso moves to affiliate itself with PC Optimum, no longer Aeroplan, in loyalty point switch""Mobil brand of gas stations to launch in Canada after deal for 213 Loblaws-owned locations""Mobil Nears Completion of Rebranding 200 Loblaw Gas Stations""Learn about ExxonMobil's operations in Egypt""Petrol and Diesel Service Stations in Egypt - Mobil"Official websiteExxon Mobil corporate websiteMobil Industrial official websiteeeeeeeeDA04275022275790-40000 0001 0860 5061n82045453134887257134887257

Frič See also Navigation menuinternal link

Identify plant with long narrow paired leaves and reddish stems Planned maintenance scheduled April 17/18, 2019 at 00:00UTC (8:00pm US/Eastern) Announcing the arrival of Valued Associate #679: Cesar Manara Unicorn Meta Zoo #1: Why another podcast?What is this plant with long sharp leaves? Is it a weed?What is this 3ft high, stalky plant, with mid sized narrow leaves?What is this young shrub with opposite ovate, crenate leaves and reddish stems?What is this plant with large broad serrated leaves?Identify this upright branching weed with long leaves and reddish stemsPlease help me identify this bulbous plant with long, broad leaves and white flowersWhat is this small annual with narrow gray/green leaves and rust colored daisy-type flowers?What is this chilli plant?Does anyone know what type of chilli plant this is?Help identify this plant